LSAT and Law School Admissions Forum

Get expert LSAT preparation and law school admissions advice from PowerScore Test Preparation.

User avatar
 Dave Killoran
PowerScore Staff
  • PowerScore Staff
  • Posts: 5853
  • Joined: Mar 25, 2011
|
#92501
Complete Question Explanation
(The complete setup for this game can be found here: lsat/viewtopic.php?f=272&t=8596)

The correct answer choice is (B)

Template #1 indicates that J, P, and N could be at Souderton. Template #2 adds O to the list. Accordingly, answer choice (B) is correct.
User avatar
 mmhubbar
  • Posts: 6
  • Joined: Jul 10, 2022
|
#96619
Hi there,

How would you go about solving this problem in the case you did not have time to make templates? Would you attack the questions in terms of if they violated certain rules, and if so, in what order would you attack them? Would you opt to use the conditional chain that this game creates?
 Rachael Wilkenfeld
PowerScore Staff
  • PowerScore Staff
  • Posts: 1370
  • Joined: Dec 15, 2011
|
#96802
Hi mmhubbar,

If you don't use templates, then this is just a list question.

Answer choice (A) would be eliminated because of the first rule (Js :arrow: Kr)
Answer choice (C) would be eliminated because we can never have Ls because of the conflicting conditionals in rules 3-5.
Answer choice (D) would be eliminated because of the last rule (Pr :arrow: Ks)
Answer choice (E) would be eliminated because of the 4th rule (Nr :arrow: Or)

It's doable without the templates, but significantly slower. It takes a bit of time to think through which are in R based on who is in S.

Hope that helps!

Get the most out of your LSAT Prep Plus subscription.

Analyze and track your performance with our Testing and Analytics Package.